Những bài Bất đẳng thức từ các cuộc thi Toán

Lời giải 3 (V. Q. B. Cẩn). Ta sẽ sử dụng phương pháp dồn biến để chứng minh bất đẳng thức đã

cho. Để ý rằng ngoài điểm đẳng thức là a = b = c = 1 thì bất đẳng thức đã cho còn có một điểm

"nhạy cảm" là a = b → 2,c → 0 (cùng các hoán vị). Điều này gợi cho ta giả sử c = min{a,b,c} và

dùng phép dồn biến để đưa hai biến a,b về bằng nhau và bằng một số t dương nào đó. Muốn vậy,

việc trước tiên ta phải làm đó là đảm bảo giả thiết của bài toán, tức là bộ số (t,t,c) phải thỏa mãn

t2 +2tc +t2c = ab +bc +ca +abc = 4. Vì ta cần dồn biến từ (a,b,c) về (t,t,c) nên ta phải chứng

minh

pdf65 trang | Chia sẻ: lethuong715 | Lượt xem: 550 | Lượt tải: 0download
Bạn đang xem trước 20 trang mẫu tài liệu Những bài Bất đẳng thức từ các cuộc thi Toán, để tải tài liệu gốc về máy bạn click vào nút DOWNLOAD ở trên
đẳng thức bên phải được chứng minh xong. Dễ thấy ở cả hai bất đẳng thức (bên trái và bên phải)
đẳng thức chỉ xảy ra tại một điểm là (a,b) = (1,1).
Bài CH4. Chứng minh rằng với mọi số thực không âm a,b,c thỏa mãn không có hai số nào trong
chúng có thể đồng thời bằng 0, bất đẳng thức sau luôn được thỏa mãn
a
a2+3bc
+
b
b2+3ca
+
c
c2+3ab
≤ (a+b+ c)
3
4(ab+bc+ ca)2
.
(Dương đức Lâm)
Lời giải (V. Q. B. Cẩn). Nhân cả hai vế của bất đẳng thức cho ab+ bc+ ca > 0, và để ý rằng
a− a(ab+bc+ca)a2+3bc =
a(a−b)(a−c)
a2+3bc +
abc
a2+3bc , ta có thể viết lại nó như sau
∑
cyc
a(a−b)(a− c)
a2+3bc
+abc∑
cyc
1
a2+3bc
+
(a+b+ c)3
4(ab+bc+ ca)
− (a+b+ c)≥ 0.
Áp dụng bất đẳng thức Cauchy Schwarz, ta có ∑
cyc
1
a2+3bc ≥ 9a2+b2+c2+3ab+3bc+3ca nên bất đẳng thức
trên được suy ra từ
∑
cyc
a(a−b)(a− c)
a2+3bc
+
9abc
a2+b2+ c2+3ab+3bc+3ca
+
(a+b+ c)3
4(ab+bc+ ca)
− (a+b+ c)≥ 0,
tương đương
∑
cyc
a(a−b)(a− c)
a2+3bc
+
(a+b+ c)3+9abc
4(ab+bc+ ca)
− (a+b+ c)≥ 9abc
4∑
cyc
ab
− 9abc
∑
cyc
a2+3∑
cyc
ab
.
Ta có (a+b+ c)3+9abc−4(a+b+ c)(ab+bc+ ca) = ∑
cyc
a(a−b)(a− c) và
a2+b2+ c2− (ab+bc+ ca) =∑
cyc
(a−b)(a− c),
nên bất đẳng thức trên tương đương với
∑
cyc
a(a−b)(a− c)
a2+3bc
+
∑
cyc
a(a−b)(a− c)
4(ab+bc+ ca)
≥
9abc∑
cyc
(a−b)(a− c)
4
(
∑
cyc
ab
)(
∑
cyc
a2+3∑
cyc
ab
) .
Ta thấy bất đẳng thức này có dạng X(a−b)(a− c)+Y (b− c)(b−a)+Z(c−a)(c−b)≥ 0, với
X = a+
4a(ab+bc+ ca)
a2+3bc
− 9abc
a2+b2+ c2+3(ab+bc+ ca)
≥ a+ 4a(ab+bc+ ca)
a2+b2+ c2+3(ab+bc+ ca)
− 9abc
a2+b2+ c2+3(ab+bc+ ca)
=
a[a2+7a(b+ c)+(b− c)2]
a2+b2+ c2+3(ab+bc+ ca)
≥ 0,
32 Let the solutions say your method - Võ Quốc Bá Cẩn
và các biểu thức Y,Z tương tự. Bây giờ, giả sử rằng a ≥ b ≥ c, ta sẽ chứng minh aX ≥ bY, tương
đương
(a2−b2)+4(ab+bc+ ca)
(
a2
a2+3bc
− b
2
b2+3ca
)
≥ 9abc(a−b)
a2+b2+ c2+3(ab+bc+ ca)
.
Một điều dễ thấy là a
2
a2+3bc ≥ b
2
b2+3ca và a
2+b2+ c2+3(ab+bc+ ca)≥ 12bc, suy ra
VT −VP≥ (a2−b2)− 9a ·
a2+b2+c2+3(ab+bc+ca)
12 · (a−b)
a2+b2+ c2+3(ab+bc+ ca)
=
(a−b)(a+4b)
4
≥ 0.
Đến đây, với để ý rằng Z(c−a)(c−b)≥ 0 và a− c≥ ab(b− c)≥ 0, ta thu được
∑
cyc
X(a−b)(a− c)≥ X(a−b)(a− c)+Y (b− c)(b−a)
≥ X(a−b) · a
b
(b− c)+Y (b− c)(b−a) = (aX−bY )(a−b)(b− c)
b
≥ 0.
Bài toán được giải quyết xong. Đẳng thức xảy ra khi và chỉ khi a = b = c hoặc (a,b,c) là một hoán
vị của bộ số (t, t,0) với t là một số dương bất kì.
Bài CH5. Cho các số thực dương a,b,c thỏa mãn ab+ bc+ ca+ abc = 4. Tìm tất cả các số thực k
sao cho bất đẳng thức sau luôn được thỏa mãn
(k+bc)(k+ ca)(k+ab)≥ (k+1)3.
(Vuonga2khtn*2)
Lời giải (V. Q. B. Cẩn). Cho c= t > 0 và a= b= 2t+1 thì ta dễ thấy ab+bc+ ca+abc= 4 và bất
đẳng thức đã cho trở thành
(
k+ 2tt+1
)2 [
k+ 4(t+1)2
]
≥ (k+1)3, tương đương
(t−1)2[(k2+ k−1)t2+(2k2−2k−6)t+ k2−3k−1]
(t+1)4
≥ 0.
Và như vậy, theo yêu cầu của đề bài, ta cần có (k2 + k− 1)t2 +(2k2− 2k− 6)t + k2− 3k− 1 ≥ 0.
Vế trái của bất đẳng thức này là một tam thức bậc 2 của t, và chúng ta đều biết rằng để nó không
âm với mọi t dương thì một điều kiện cần là hai hệ số cao nhất và thấp nhất phải không âm, tức là
k2+k−1≥ 0 và k2−3k−1≥ 0. Từ đây, ta tìm được k≤−1+
√
5
2 hoặc k≥ 3+
√
13
2 . Ta sẽ chứng minh
đây chính là tập hợp tất cả các giá trị cần tìm của k, tức là
(k+bc)(k+ ca)(k+ab)≥ (k+1)3.
Để chứng minh, chúng ta sẽ chia làm 2 trường hợp
+ Xét k ≥ 3+
√
13
2 . Khi đó, áp dụng kết quả bài O2, ta có
(k+bc)(k+ ca)(k+ab) = k3+ k2(ab+bc+ ca)+ kabc(a+b+ c)+a2b2c2
≥ k3+ k2(ab+bc+ ca)+ kabc(ab+bc+ ca)+a2b2c2
= k3+ k2(4−abc)+ kabc(4−abc)+a2b2c2.
2Những bài mà chúng tôi không biết tên thật của tác giả và chỉ biết nickname, chúng tôi sẽ ghi nickname kèm theo dấu
* ở phía sau. Khi nào biết được tên thật sự của tác giả, chúng tôi xin sửa lại và ghi đúng tên của người đặt ra bài toán.
Những bài bất đẳng thức tự sáng tạo và sưu tầm 33
Mà k3+ k2(4−abc)+ kabc(4−abc)+a2b2c2− (k+1)3 = (1−abc)[(k−1)abc+ k2−3k−1]≥ 0
(do 1≥ abc (đánh giá này được suy ra trực tiếp từ giả thiết), (k−1)abc≥ 0 và k2−3k−1≥ 0) nên
hiển nhiên
(k+bc)(k+ ca)(k+ab)≥ (k+1)3.
+ Xét k ≤−1+
√
5
2 <−1. Đặt
√
ab = x,
√
bc = y,
√
ca = z thì ta có x2+ y2+ z2+ xyz = 4, và ta phải
chứng minh (k+ x2)(k+ y2)(k+ z2)≥ (k+1)3. Áp dụng bất đẳng thức Schur bậc 3, ta có
4(x2y2+ y2z2+ z2x2)≤ 9x
2y2z2
x2+ y2+ z2
+(x2+ y2+ z2)2
=
9t2
4− t +(4− t)
2 (t = xyz≤ 1).
Từ đó suy ra
(k+ x2)(k+ y2)(k+ z2)− (k+1)3 = k2(x2+ y2+ z2−3)+ k(x2y2+ y2z2+ z2x2−3)+ t2−1
≥ k2(1− t)+ k
[
9t2
4−t +(4− t)2
4
−3
]
+ t2−1
= (1− t)
[
k2+
k(t2−20t+16)
4(4− t) − t−1
]
.
Lại có
k(t2−20t+16)
4(4− t) − t− k =
t[(k+4)t−16(k+1)]
4(4− t) ≥
t[(k+4)t−16(k+1)t]
4(4− t) =−
3t2(5k+4)
4(4− t) ≥ 0,
nên
k2+
k(t2−20t+16)
4(4− t) − t−1≥ k
2+ k−1≥ 0.
Như vậy, khẳng định của ta đã được chứng minh xong. Và do đó, tập hợp tất cả các giá trị của k thỏa
mãn yêu cầu của đề bài là k ∈
(
−∞,−1+
√
5
2
]
∪
[
3+
√
13
2 ,+∞
)
.
Bài CH6. Cho a,b,c,d là các số thực không âm thỏa mãn a2+b2+ c2+d2 = 1. Chứng minh rằng
a3+b3+ c3+d3+8(1−a)(1−b)(1− c)(1−d)≥ 1.
(Phạm Văn Thuận)
Lời giải (V. Q. B. Cẩn). Chúng tôi xin được giới thiệu cùng bạn đọc chứng minh sau. Mặc dù là một
chứng minh không đẹp nhưng nó lại là một ý tưởng mới về bất đẳng thức (chuyển từ bất đẳng thức
thuần nhất sang dạng không thuần nhất). Từ giả thiết, ta dễ dàng suy ra được a,b,c,d ∈ [0,1] và ta
cũng có thể viết lại bất đẳng thức cần chứng minh dưới dạng P(a,b,c,d)≥ 0, trong đó
P(a,b,c,d) =∑
cyc
a3+
1
4∑cyc
a2+8(1−a)(1−b)(1− c)(1−d)− 5
4
.
Bây giờ, ta sẽ chứng minh bất đẳng thức P(a,b,c,d)≥ 0 đúng với mọi a,b,c,d ∈ [0,1]mà không cần
thiết phải thỏa mãn a2+b2+ c2+d2 = 1. Thật vậy, ta có
P(a,b,c,d)−P
(
a+b
2
,
a+b
2
,c,d
)
=
(a−b)2[6(a+b)+1−16(1− c)(1−d)]
8
,
P(a,b,c,d)−P(a+b,0,c,d) =−ab[6(a+b)+1−16(1− c)(1−d)]
2
,
P(a,b,c,d)−P(a+b−1,1,c,d) =−(1−a)(1−b)[6(a+b)+1−16(1− c)(1−d)]
2
.
34 Let the solutions say your method - Võ Quốc Bá Cẩn
Từ đây, ta thấy nếu 6(a+ b) + 1− 16(1− c)(1− d) ≥ 0 thì P(a,b,c,d) ≥ P( a+b2 , a+b2 ,c,d) . Nếu
6(a+b)+1−16(1− c)(1−d) ≤ 0 và a+b ≤ 1 thì P(a,b,c,d) ≥ P(a+b,0,c,d). Nếu 6(a+b)+
1− 16(1− c)(1− d) ≤ 0 và a+ b ≥ 1 thì P(a,b,c,d) ≥ P(a+ b− 1,1,c,d). Những lập luận này
chứng tỏ rằng, để chứng minh bất đẳng thức đã cho, ta chỉ cần chứng minh nó đúng trong ba trường
hợp sau là đủ a = b,ab = 0 và (1− a)(1− b) = 0. Hoàn toàn tương tự, ta cũng thấy rằng chỉ cần
chứng minh bất đẳng thức đúng trong ba trường hợp c= d,cd = 0 và (1− c)(1−d) = 0 thì bài toán
cũng được giải quyết xong. Kết hợp hai lập luận này lại và loại bớt những trường hợp trùng nhau, ta
có thể đưa bài toán về xét trong 4 trường hợp sau
+ Xét a= b và c= d. Khi đó, bất đẳng thức cần chứng minh trở thành
2a3+2c3+
a2+ c2
2
+8(1−a)2(1− c)2− 5
4
≥ 0.
Đặt t = a+ c và x= ac
(
0≤ x≤ t24
)
, ta có thể viết lại nó như sau
2t(t2−3x)+ t
2−2x
2
+8(1− t+ x)2− 5
4
≥ 0,
tương đương
f (x) = 32x2+4(15−22t)x+8t3+34t2−64t+27≥ 0.
Nếu 4t2−22t+15≤ 0 thì ta có f ′(x) = 64x+4(15−22t)≤ 16t2+4(15−22t) = 4(4t2−22t+15)≤
0, dẫn đến f (x) là hàm giảm với mọi x≤ t24 , và ta thu được
f (x)≥ f
(
t2
4
)
= (2t2−10t+27)(t−1)2 ≥ 0.
Nếu 15−22t ≥ 0 thì bất đẳng thức là hiển nhiên đúng bởi vì ta luôn có 8t3+34t2−64t+27> 0 với
mọi t ≥ 0.
Nếu 15−22t ≤ 0 và 4t2−22t+15≥ 0 thì ta có 1522 ≤ t ≤ 11−
√
61
4 , khi đó dễ thấy
∆′f = 4(15−22t)2−32(8t3+34t2−64t+27) =−4(64t3−212t2+148t−9)< 0,
nên f (x) luôn đạt giá trị không âm với mọi a,c ∈ [0,1]. Trường hợp thứ nhất được giải quyết xong.
+ Xét a= b và d = 0. Khi đó, bất đẳng thức cần chứng minh trở thành
2a3+ c3+
2a2+ c2
4
+8(1−a)2(1− c)− 5
4
≥ 0,
tương đương
2a3+
a2
2
−8a(2−a)(1− c)+ (1− c)(27−5c−4c
2)
4
≥ 0.
Do 1− c≥ 0 và 27−5c−4c2 ≥ 27−9c nên ta chỉ cần chứng minh được
2a3+
a2
2
−8a(2−a)(1− c)+ 9(1− c)(3− c)
4
≥ 0,
hay là
f (c) = 9c2−4(8a2−16a+9)c+8a3+34a2−64a+27≥ 0.
Nếu 16a2−32a+9≥ 0 thì ta có f ′(c)= 18c−4(8a2−16a+9)≤ 18−4(8a2−16a+9)=−2(16a2−
32a+9)≤ 0 nên f (c) là hàm giảm với mọi c≤ 1, và ta thu được f (c)≥ f (1) = 2a2(4a+1)≥ 0.
Những bài bất đẳng thức tự sáng tạo và sưu tầm 35
Nếu 16a2−32a+9≤ 0 thì 4−
√
7
4 ≤ a≤ 1, khi đó bằng cách tính biệt thức của f (c), ta dễ thấy
∆′f = 4(8a
2−16a+9)2−9(8a3+34a2−64a+27) = 256a4−1096a3+1294a2−576a+81< 0,
dẫn đến f (c)≥ 0 và trường hợp thứ hai cũng được giải quyết xong.
+ Xét b= 1. Khi đó, bất đẳng thức cần chứng minh trở thành a3+ c3+d3+ a
2+c2+d2
4 ≥ 0, là một bất
đẳng thức hiển nhiên đúng do a,c,d là những số không âm.
+ Xét b= 0 và d = 0. Khi đó, ta phải chứng minh
a3+ c3+
a2+ c2
4
+8(1−a)(1− c)− 5
4
≥ 0.
Đặt t = a+ c và x= ac
(
0≤ x≤ t24
)
, bất đẳng thức này trở thành
t(t2−3x)+ t
2−2x
4
+8(1− t+ x)− 5
4
≥ 0,
hay là
6(5−2t)x+(1− t)(27−5t−4t2)≥ 0.
Nếu t ≤ 1 thì bất đăng thức cuối là hiển nhiên bởi vì ta có 6(5−2t)x≥ 0 và (1−t)(27−5t−4t2)≥ 0.
Trong trường hợp ngược lại, sử dụng đánh giá (1−a)(1− c)≥ 0, ta suy ra được x≥ t−1, dẫn đến
6(5−2t)x+(1− t)(27−5t−4t2)≥ 6(5−2t)(t−1)+(1− t)(27−5t−4t2) = (4t−3)(t−1)2 ≥ 0.
Trường hợp thứ tư cũng được giải quyết xong. Và như thế, phép chứng minh của ta cũng được hoàn
tất. Dễ thấy đẳng thức xảy ra khi và chỉ khi a= b= c= d = 12 hoặc a= 1,b= c= d = 0 và các hoán
vị tương ứng.
Bài CH7. Cho các số thực dương a,b,c thỏa mãn a+ b+ c = a3 + b3 + c3. Chứng minh bất đẳng
thức sau
a
a2+1
·
( c
b
)2
+
b
b2+1
·
(a
c
)2
+
c
c2+1
·
(
b
a
)2
≥ a+b+ c
2
.
(Gabriel Dospinescu)
Lời giải (V. Q. B. Cẩn). Áp dụng bất đẳng thức Cauchy Schwarz, ta dễ thấy
VT ·
(
a2+1
a
+
b2+1
b
+
c2+1
c
)
≥
(
c
b
+
a
c
+
b
a
)2
.
Do đó, ta chỉ cần chứng minh được
2
(
c
b
+
a
c
+
b
a
)2
≥ (a+b+ c)
(
a+b+ c+
1
a
+
1
b
+
1
c
)
.
Ta thấy rằng bất đẳng thức này chính là tổng của hai bất đẳng thức sau(
c
b
+
a
c
+
b
a
)2
≥ (a+b+ c)
(
1
a
+
1
b
+
1
c
)
,
và (
c
b
+
a
c
+
b
a
)2
≥ (a+b+ c)2.
36 Let the solutions say your method 

File đính kèm:

  • pdfTL BDT hot cua ban Ba Can dayTuan AnhNga Dien.pdf